Mathcenter Forum  

Go Back   Mathcenter Forum > คณิตศาสตร์โอลิมปิก และอุดมศึกษา > ทฤษฎีจำนวน
สมัครสมาชิก คู่มือการใช้ รายชื่อสมาชิก ปฏิทิน ข้อความวันนี้

ตั้งหัวข้อใหม่ Reply
 
เครื่องมือของหัวข้อ ค้นหาในหัวข้อนี้
  #76  
Old 05 มกราคม 2006, 23:26
gools's Avatar
gools gools ไม่อยู่ในระบบ
บัณฑิตฟ้า
 
วันที่สมัครสมาชิก: 26 เมษายน 2004
ข้อความ: 390
gools is on a distinguished road
Post

23. หาคำตอบที่เป็นจำนวนเต็มของสมการ
\[x^3+y^3+z^3=500\]

Edit: ใส่เลขข้อ

22 กุมภาพันธ์ 2007 14:54 : ข้อความนี้ถูกแก้ไขแล้ว 1 ครั้ง, ครั้งล่าสุดโดยคุณ warut
ตอบพร้อมอ้างอิงข้อความนี้
  #77  
Old 06 มกราคม 2006, 07:06
nongtum's Avatar
nongtum nongtum ไม่อยู่ในระบบ
ผู้พิทักษ์กฎทั่วไป
 
วันที่สมัครสมาชิก: 10 เมษายน 2005
ข้อความ: 3,246
nongtum is on a distinguished road
Icon16

อ้างอิง:
ข้อความเดิมของคุณ gools:
23. หาคำตอบที่เป็นจำนวนเต็มของสมการ
\[x^3+y^3+z^3=500\]
เริ่มจากสังเกตว่าผลรวมของสามจำนวนเต็มใดจะเป็นเลขคู่ก็ต่อเมื่อ i)ทั้งสามตัวเป็นเลขคู่ หรือ ii)เป็นเลขคี่สองตัว เลขคู่หนึ่งตัว และเศษที่ได้จากการหารกำลังสามของจำนวนเต็มใดด้วยสามจะเท่ากับเศษที่ได้จากการหารจำนวนเต็มนั้นด้วยสาม และ $500\equiv2\bmod{3}$ ดังนั้นจึงแจงกรณีได้ดังนี้ (จำนวนเต็ม a,b,c ในแต่ละกรณีไม่จำเป็นต้องเท่ากัน)
$\begin{array}{llclll}
ก) x,y,z คู่,& x,y\equiv0(3),\ z\equiv2(3) &\Rightarrow& x=6a,& y=6b,& z=6c+2\\
ข) x,y,z คู่,& x,y\equiv1(3),\ z\equiv0(3)&\Rightarrow& x=6a+4,& y=6b+4,& z=6c\\
ค) x,y,z คู่,& x,y\equiv2(3),\ z\equiv1(3)&\Rightarrow& x=6a+2,& y=6b+2,& z=6c+4\\
ง) x คู่, y,z คี่,& x,y\equiv0(3),\ z\equiv2(3)&\Rightarrow& x=6a,& y=6b+3,& z=6c+5\\
จ) x คู่, y,z คี่,& x,y\equiv1(3),\ z\equiv0(3)&\Rightarrow& x=6a+4,& y=6b+1,& z=6c+3\\
ฉ) x คู่, y,z คี่,& x,y\equiv2(3),\ z\equiv1(3)&\Rightarrow& x=6a+2,& y=6b+5,& z=6c+1\\
\end{array}$
ทุกกรณี (แสดงเฉพาะกรณี ก ที่เหลือทำคล้ายๆกัน) แทน x,y,z ในสมการโจทย์ รวมเทอมแล้วลบด้วยจำนวนเต็มที่ไม่ติดตัวแปรจะได้ $(6a)^3+(6b)^3+(6c+2)^3-2^3=492$ ซึ่ง 18 หารผลรวมทางซ้ายมือลงตัว แต่หารทางขวามือไม่ลงตัว
ดังนั้นจึงสรุปได้ว่าสมการที่ให้ไม่มีคำตอบเป็นจำนวนเต็ม###

งวดนี้ลองโจทย์ที่ยากกว่าเดิมหน่อยละกัน
24. (KöMaL ข้อ N.149) The sequence $(a_n)$ is defined by the following recursion: $$a_0=a_1=1,\ (n+1)a_{n+1}=(2n+1)a_n+3na_{n-1}.$$ Prove that the sequence consists of integers.
__________________
คนไทยร่วมใจอย่าใช้ภาษาวิบัติ
ฝึกพิมพ์สัญลักษณ์สักนิด ชีวิต(คนตอบและคนถาม)จะง่ายขึ้นเยอะ (จริงๆนะ)

Stay Hungry. Stay Foolish.
ตอบพร้อมอ้างอิงข้อความนี้
  #78  
Old 06 มกราคม 2006, 18:28
warut warut ไม่อยู่ในระบบ
กระบี่ไร้สภาพ
 
วันที่สมัครสมาชิก: 24 พฤศจิกายน 2001
ข้อความ: 1,627
warut is on a distinguished road
Smile

อ้างอิง:
ข้อความเดิมของคุณ gools:
23. หาคำตอบที่เป็นจำนวนเต็มของสมการ
\[x^3+y^3+z^3=500\]
เนื่องจากถ้า \(a\) เป็นจำนวนเต็มแล้ว \(a^3\equiv0,\pm1\pmod9\) และ \(500\equiv-4\pmod9\) ดังนั้นสมการโจทย์จึงไม่มีคำตอบครับ

ป.ล. อยากจะขอร้องน้อง gools และเพื่อนๆสมาชิกคนอื่นๆว่าหากโพสต์โจทย์ไปแล้วก็ช่วย "follow up" ด้วยครับ เช่น ถ้ามีใครถามไถ่เพิ่มเติมก็ตอบให้เขาหน่อย หรือถ้าเวลาผ่านไปนานแล้วแต่ยังไม่มีใครให้คำตอบที่ดีๆก็ช่วยเฉลยให้ด้วย ฯลฯ ผมว่าถ้าทำเช่นนี้ได้ที่นี่ก็จะได้เป็นที่แลกเปลี่ยนความรู้ทางคณิตศาสตร์อย่างแท้จริงของพวกเราครับ

22 กุมภาพันธ์ 2007 14:56 : ข้อความนี้ถูกแก้ไขแล้ว 2 ครั้ง, ครั้งล่าสุดโดยคุณ warut
ตอบพร้อมอ้างอิงข้อความนี้
  #79  
Old 07 มกราคม 2006, 02:55
Punk Punk ไม่อยู่ในระบบ
ลมปราณบริสุทธิ์
 
วันที่สมัครสมาชิก: 10 เมษายน 2005
ข้อความ: 108
Punk is on a distinguished road
Post

24. (Easy and Tricky) จากเงื่อนไขเริ่มทำให้ได้ว่า $b_n:=na_n$ ต้องเป็นจำนวนนับ (ทำไม??? ) แปลงสมการในเทอมของ ลำดับ $\{b_n\}$ ได้เป็น
$$
(b_{n+1}-2b_n-3b_{n-1})n^2-(b_{n+1}-b_n)n+b_n=0
$$
ดังนั้น $n$ หาร $b_n$ ลงตัว

07 มกราคม 2006 06:01 : ข้อความนี้ถูกแก้ไขแล้ว 1 ครั้ง, ครั้งล่าสุดโดยคุณ Punk
ตอบพร้อมอ้างอิงข้อความนี้
  #80  
Old 08 มกราคม 2006, 15:53
gools's Avatar
gools gools ไม่อยู่ในระบบ
บัณฑิตฟ้า
 
วันที่สมัครสมาชิก: 26 เมษายน 2004
ข้อความ: 390
gools is on a distinguished road
Post

25. จงแสดงว่า $\frac{a^2-2}{2b^2+3}$ ไม่เป็นจำนวนเต็ม เมื่อ $a,b$ เป็นจำนวนเต็ม
ตอบพร้อมอ้างอิงข้อความนี้
  #81  
Old 16 มกราคม 2006, 06:03
nongtum's Avatar
nongtum nongtum ไม่อยู่ในระบบ
ผู้พิทักษ์กฎทั่วไป
 
วันที่สมัครสมาชิก: 10 เมษายน 2005
ข้อความ: 3,246
nongtum is on a distinguished road
Post

เนื่องจากข้อ 25. ถูกนำไปเล่นเป็นโจทย์ปัญหาชิงรางวัลข้อที่ 5ไปแล้ว ขอตั้งโจทย์ข้อถัดไปไม่ยากมากให้กระทู้เดินละกันนะครับ

26. ให้ n เป็นจำนวนนับ จงแสดงว่า $$256^{2n}\cdot7^{2n}-168^{2n}-32^{2n}+3^{2n}$$ หารด้วย 1995 ลงตัว (ใบ้: $1995=3\cdot5\cdot7\cdot19$)
__________________
คนไทยร่วมใจอย่าใช้ภาษาวิบัติ
ฝึกพิมพ์สัญลักษณ์สักนิด ชีวิต(คนตอบและคนถาม)จะง่ายขึ้นเยอะ (จริงๆนะ)

Stay Hungry. Stay Foolish.

06 เมษายน 2007 09:05 : ข้อความนี้ถูกแก้ไขแล้ว 1 ครั้ง, ครั้งล่าสุดโดยคุณ gon
เหตุผล: Tag Post
ตอบพร้อมอ้างอิงข้อความนี้
  #82  
Old 17 มกราคม 2006, 10:50
Coco's Avatar
Coco Coco ไม่อยู่ในระบบ
หัดเดินลมปราณ
 
วันที่สมัครสมาชิก: 23 กรกฎาคม 2005
ข้อความ: 48
Coco is on a distinguished road
Post

เนื่องจาก

$ 256^{2n} \cdot 7^{2n} - 168^{2n} - 32^{2n} + 3^{2n} \equiv 1^{2n} \cdot 1^{2n} - 0^{2n} - (-1)^{2n} + 0^{2n} \equiv 1 - 0 - 1 + 0 \equiv 0 (mod 3)$
$ 256^{2n} \cdot 7^{2n} - 168^{2n} - 32^{2n} + 3^{2n} \equiv 1^{2n} \cdot 2^{2n} - (-2)^{2n} - 2^{2n} + (-2)^{2n} \equiv 4^n - 4^n - 4^n + 4^n \equiv 0 (mod 5) $
$ 256^{2n} \cdot 7^{2n} - 168^{2n} - 32^{2n} + 3^{2n} \equiv 4^{2n} \cdot 0^{2n} - 0^{2n} - (-3)^{2n} + 3^{2n} \equiv 0 - 0 - 9^n + 9^n \equiv 0 (mod 7) $
$ 256^{2n} \cdot 7^{2n} - 168^{2n} - 32^{2n} + 3^{2n} \equiv 9^{2n} \cdot 7^{2n} - (-3)^{2n} - (-6)^{2n} + 3^{2n} \equiv 36^n - 9^n - 36^n + 9^n \equiv 0 (mod 19) $

และ 3, 5, 7, 19 เป็น pairwise relatively prime ดังนั้น

$ 256^{2n} \cdot 7^{2n} - 168^{2n} - 32^{2n} + 3^{2n} \equiv 0(mod 3 \cdot 5 \cdot 7 \cdot 19) $
__________________
สนใจคณิตศาสตร์ครับ ช่วยชี้แนะด้วยครับ
ตอบพร้อมอ้างอิงข้อความนี้
  #83  
Old 17 มกราคม 2006, 11:22
nongtum's Avatar
nongtum nongtum ไม่อยู่ในระบบ
ผู้พิทักษ์กฎทั่วไป
 
วันที่สมัครสมาชิก: 10 เมษายน 2005
ข้อความ: 3,246
nongtum is on a distinguished road
Smile

วิธีทำข้างบนถูกครับ แต่มีข้อเสนอแนะเกี่ยวกับการเขียนวิธีทำดังนี้

1. ไม่จำเป็นต้องเขียนเทอมซ้ำกันทุกบรรทัดครับ เพราะหากโจทย์มีวิธีทำยาวๆ อ่านแล้วชวนลายตาและลืมได้ง่ายๆว่ากำลังพูดถึงอะไร
หากอยากให้อ่านง่ายๆหน่อยอาจเขียนว่า "ให้ A=...(เทอมยาวๆเขียนยากๆที่หาหรือใช้บ่อยๆ)" แล้วต่อมาก็ใช้แต่ A ครับ
2. ตรงการแสดงคำนวณเขียนอธิบายเพิ่มหน่อยก็ดีครับ คนอ่านจะได้อยากอ่านวิธีทำ(ไม่ใช่กระดาษทด)
3. หากจะพิมพ์ $p\pmod a$ ให้พิมพ์ p\pmod a หรือ p\pmod{a} ครับ แล้ววงเล็บครอบจะตามมาเอง

คุณ coco มีสิทธิ์ตั้งโจทย์ข้อถัดไปครับ
__________________
คนไทยร่วมใจอย่าใช้ภาษาวิบัติ
ฝึกพิมพ์สัญลักษณ์สักนิด ชีวิต(คนตอบและคนถาม)จะง่ายขึ้นเยอะ (จริงๆนะ)

Stay Hungry. Stay Foolish.

17 มกราคม 2006 11:31 : ข้อความนี้ถูกแก้ไขแล้ว 1 ครั้ง, ครั้งล่าสุดโดยคุณ nongtum
ตอบพร้อมอ้างอิงข้อความนี้
  #84  
Old 18 มกราคม 2006, 08:53
Coco's Avatar
Coco Coco ไม่อยู่ในระบบ
หัดเดินลมปราณ
 
วันที่สมัครสมาชิก: 23 กรกฎาคม 2005
ข้อความ: 48
Coco is on a distinguished road
Post

ขอลองตั้งโจทย์ดูบ้างครับ

27. จงแสดงว่าสำหรับจำนวนเต็ม a, b, c และ d ถ้า (a,b) = 1 และ (c,d) = 1 และ $ \frac{a}{b} + \frac{c}{d} $ เป็นจำนวนเต็ม แล้ว |b| = |d|
__________________
สนใจคณิตศาสตร์ครับ ช่วยชี้แนะด้วยครับ
ตอบพร้อมอ้างอิงข้อความนี้
  #85  
Old 18 มกราคม 2006, 14:13
warut warut ไม่อยู่ในระบบ
กระบี่ไร้สภาพ
 
วันที่สมัครสมาชิก: 24 พฤศจิกายน 2001
ข้อความ: 1,627
warut is on a distinguished road
Smile

ข้อ 27. นี่สามารถทำได้ด้วยวิธีที่ผมทำข้อ 21. ของข้อสอบโอลิมปิกไทยปี 45 โดยอาศัยความจริงที่ว่า \(x|y\Rightarrow|x|\le|y|\) ครับ

06 เมษายน 2007 09:08 : ข้อความนี้ถูกแก้ไขแล้ว 1 ครั้ง, ครั้งล่าสุดโดยคุณ gon
เหตุผล: Tag Post
ตอบพร้อมอ้างอิงข้อความนี้
  #86  
Old 18 มกราคม 2006, 18:32
Coco's Avatar
Coco Coco ไม่อยู่ในระบบ
หัดเดินลมปราณ
 
วันที่สมัครสมาชิก: 23 กรกฎาคม 2005
ข้อความ: 48
Coco is on a distinguished road
Post

พี่ Warut ครับ ผมขอตั้งโจทย์ข้อต่อไปนะครับ

28. จงหาจำนวนตรรกยะ x และ y ทั้งหมดที่ทำให้ x2 + y2 = 1
__________________
สนใจคณิตศาสตร์ครับ ช่วยชี้แนะด้วยครับ
ตอบพร้อมอ้างอิงข้อความนี้
  #87  
Old 18 มกราคม 2006, 19:01
nongtum's Avatar
nongtum nongtum ไม่อยู่ในระบบ
ผู้พิทักษ์กฎทั่วไป
 
วันที่สมัครสมาชิก: 10 เมษายน 2005
ข้อความ: 3,246
nongtum is on a distinguished road
Smile

โดยใช้ผลจากข้อ 27 เราสามารถสมมติโดยไม่เสียนัยว่าให้ x=a/c, y=b/c สำหรับจำนวนเต็ม a,b,c, c0, (a,c)=(b,c)=1
แล้ว $\frac{a^2}{c^2}+\frac{b^2}{c^2}=1\quad\Leftrightarrow\quad a^2+b^2=c^2$
ซึ่งมี pythagoras triple $(a,b,c)=(m^2-n^2,2mn,m^2+n^2),\ m,n\in\mathbb{N},\ m\ge n$ เป็นคำตอบ
ดังนั้น $x=\pm\frac{m^2-n^2}{m^2+n^2},\ y=\pm\frac{2mn}{m^2+n^2},\ m,n\in\mathbb{N}$ เป็นจำนวนตรรกยะทั้งหมดที่สอดคล้องเงื่อนไขโจทย์ (ค่าของ x,y สลับกันได้)
__________________
คนไทยร่วมใจอย่าใช้ภาษาวิบัติ
ฝึกพิมพ์สัญลักษณ์สักนิด ชีวิต(คนตอบและคนถาม)จะง่ายขึ้นเยอะ (จริงๆนะ)

Stay Hungry. Stay Foolish.

18 มกราคม 2006 19:19 : ข้อความนี้ถูกแก้ไขแล้ว 2 ครั้ง, ครั้งล่าสุดโดยคุณ nongtum
ตอบพร้อมอ้างอิงข้อความนี้
  #88  
Old 18 มกราคม 2006, 22:03
Coco's Avatar
Coco Coco ไม่อยู่ในระบบ
หัดเดินลมปราณ
 
วันที่สมัครสมาชิก: 23 กรกฎาคม 2005
ข้อความ: 48
Coco is on a distinguished road
Post

ข้อต่อไปนะ

29. จงหาจำนวนตรรกยะ x และ y ทั้งหมดที่ทำให้ x2 + y2 = 7
__________________
สนใจคณิตศาสตร์ครับ ช่วยชี้แนะด้วยครับ
ตอบพร้อมอ้างอิงข้อความนี้
  #89  
Old 18 มกราคม 2006, 23:42
nongtum's Avatar
nongtum nongtum ไม่อยู่ในระบบ
ผู้พิทักษ์กฎทั่วไป
 
วันที่สมัครสมาชิก: 10 เมษายน 2005
ข้อความ: 3,246
nongtum is on a distinguished road
Smile

โดยสมมติ x,y,a,b,c เหมือนวิธีทำข้อ 28 จะได้ $a^2+b^2=7c^2$
แต่จาก $a^2+b^2\equiv 0,1,2\pmod4$ และ $7c^2\equiv0,-1\pmod4$ จะได้ว่า 2 หาร a,b,c ลงตัว เกิดข้อขัดแย้งกับ (a,c)=(b,c)=1 ดังนั้นสมการที่กำหนดให้ในโจทย์จึงไม่มีคำตอบเป็นจำนวนตรรกยะ

ข้อถัดไป

30. ลำดับ $a_0,\ a_1,\ldots$ นิยามโดยความสัมพันธ์เวียนบังเกิด $$a_0=2006,\qquad a_n=na_{n-1}+1\quad สำหรับ\ n\ge1$$ จงหาเศษที่ได้จากการหาร $a_{2549}$ ด้วย 9

Edit: มาแก้เลขให้เข้ายุค
__________________
คนไทยร่วมใจอย่าใช้ภาษาวิบัติ
ฝึกพิมพ์สัญลักษณ์สักนิด ชีวิต(คนตอบและคนถาม)จะง่ายขึ้นเยอะ (จริงๆนะ)

Stay Hungry. Stay Foolish.

18 มกราคม 2006 23:51 : ข้อความนี้ถูกแก้ไขแล้ว 1 ครั้ง, ครั้งล่าสุดโดยคุณ nongtum
ตอบพร้อมอ้างอิงข้อความนี้
  #90  
Old 19 มกราคม 2006, 08:15
Coco's Avatar
Coco Coco ไม่อยู่ในระบบ
หัดเดินลมปราณ
 
วันที่สมัครสมาชิก: 23 กรกฎาคม 2005
ข้อความ: 48
Coco is on a distinguished road
Post



a2547 = 2547a2546 + 1
a2548 = 25482547a2546 + 2548 + 1
a2549 = 254925482547a2546 + 25492548 + 2549 + 1

เนื่องจาก 2547 หารด้วย 9 ลงตัวด้งนั้น a2549 0 + 21 + 2 + 1 5 (mod 9)
__________________
สนใจคณิตศาสตร์ครับ ช่วยชี้แนะด้วยครับ
ตอบพร้อมอ้างอิงข้อความนี้
ตั้งหัวข้อใหม่ Reply


หัวข้อคล้ายคลึงกัน
หัวข้อ ผู้ตั้งหัวข้อ ห้อง คำตอบ ข้อความล่าสุด
ปัญหาชิงรางวัลข้อที่ 23: Number Theory once more warut คณิตศาสตร์อุดมศึกษา 17 28 ธันวาคม 2011 20:38
ช่วยคิดหน่อยครับ เกี่ยวกับ Number Theory kanji ทฤษฎีจำนวน 0 08 กันยายน 2006 18:22
ปัญหาชิงรางวัลข้อที่ 5: From Number Theory Marathon warut คณิตศาสตร์อุดมศึกษา 9 17 มกราคม 2006 18:47
ปัญหา Number Theory kanji ทฤษฎีจำนวน 4 16 พฤศจิกายน 2005 20:30
ขอลองตั้งคำถามบ้างครับ (Number theory) Nay ทฤษฎีจำนวน 3 15 พฤษภาคม 2005 13:40


กฎการส่งข้อความ
คุณ ไม่สามารถ ตั้งหัวข้อใหม่ได้
คุณ ไม่สามารถ ตอบหัวข้อได้
คุณ ไม่สามารถ แนบไฟล์และเอกสารได้
คุณ ไม่สามารถ แก้ไขข้อความของคุณเองได้

vB code is On
Smilies are On
[IMG] code is On
HTML code is Off
ทางลัดสู่ห้อง


เวลาที่แสดงทั้งหมด เป็นเวลาที่ประเทศไทย (GMT +7) ขณะนี้เป็นเวลา 04:54


Powered by vBulletin® Copyright ©2000 - 2024, Jelsoft Enterprises Ltd.
Modified by Jetsada Karnpracha